1
$\begingroup$

Im working with the splitting lemma right now and I don't understand it too much. If someone has that book and could help me complete the sketch in page 177, proof of 1.18) I would be very grateful.

the book can be found here also http://books.google.se/books?id=t6N_tOQhafoC&printsec=frontcover&hl=sv&source=gbs_atb#v=onepage&q&f=false

  • 0
    @HagenvonEitzen Please consider converting your comment into an answer, so that this question gets removed from the [unanswered tab](http://meta.math.stackexchange.com/q/3138). If you do so, it is helpful to post it to [this chat room](http://chat.stackexchange.com/rooms/9141) to make people aware of it (and attract some upvotes). For further reading upon the issue of too many unanswered questions, see [here](http://meta.stackexchange.com/q/143113), [here](http://meta.math.stackexchange.com/q/1148) or [here](http://meta.math.stackexchange.com/a/9868).2013-09-14

1 Answers 1

1

The real question was : Was does "the diagram commutes" mean?

A diagram with a mesh with vertices $A,B,C,D$ surrounded by arrows $A\stackrel \alpha\to B\stackrel \beta\to C$ and $A\stackrel \gamma\to D\stackrel \delta\to C$ is said to commute if $δ\circ γ=β\circ α$. In the diagrams with short exact sequences, you have two such meshes; the condition just described shall hold for both meshes.